The limit of a power as it tends to infinity.

Click For Summary
SUMMARY

The limit of the expression \(\lim_{p\to\infty} \biggl(\frac{{| d |}^{np+p-1} |x|^{p-1}}{(p-1)!} \{(|x| + |\alpha_1|) \ldots (|x| + |\alpha_n|) \}^p\biggr)\) approaches zero as \(p\) tends to infinity. The factorial in the denominator grows faster than the polynomial terms in the numerator, confirming that the limit converges to zero. A suggested approach involves transforming the expression into a form resembling \(\frac{C^{Kp}}{p!}\), where \(C\) and \(K\) are constants. This method simplifies the analysis and confirms the limit's behavior.

PREREQUISITES
  • Understanding of limits in calculus
  • Familiarity with factorial growth rates
  • Knowledge of polynomial and exponential functions
  • Experience with the squeeze theorem in mathematical proofs
NEXT STEPS
  • Study the behavior of factorial functions compared to polynomial functions
  • Learn about the squeeze theorem and its applications in limit proofs
  • Explore asymptotic analysis techniques in calculus
  • Investigate the properties of exponential growth in mathematical limits
USEFUL FOR

Mathematicians, students studying calculus, and anyone interested in advanced limit proofs and asymptotic behavior in mathematical analysis.

HTale
Messages
1
Reaction score
0
I've come across a snag in a proof, and I've become a little exasperated by the following limit:


\displaystyle \lim_{p\to\infty} \biggl(\frac{{| d |}^{np+p-1} |x|^{p-1}}{(p-1)!} \{(|x| + |\alpha_1|) \ldots (|x| + |\alpha_n|) \}^p\biggr)

I've tried the squeeze rule, but an upper bound eludes me. I've only found the lower bound, which is just 0. I've tried to group together all of the powers of p, as follows:


\displaystyle \lim_{p\to\infty} \biggl(\frac{{| d x|}^{p-1}}{(p-1)!} \{|d|^n(|x| + |\alpha_1|) \ldots (|x| + |\alpha_n|) \}^p\biggr)

and maybe use the fact that

\displaystyle \frac{{| d x|}^{p-1}}{(p-1)!}​

tends to zero as p tends to infinity, since the bottom half rises faster than the top half. However, I'm stuck because I don't know how to go about this formally, or if this is the right way to go about it. I'd be really grateful if help was provided, and possibly a solution.

Thank you very very much in advance.

HTale
 
Physics news on Phys.org
There's nothing very delicate about that limit. C^(Kp)/p! goes to zero for any choice of C or K. Try to change it into a form like that. Start by picking M>|x|,|d|,|a1|,|a2|,...|an|.
 
Question: A clock's minute hand has length 4 and its hour hand has length 3. What is the distance between the tips at the moment when it is increasing most rapidly?(Putnam Exam Question) Answer: Making assumption that both the hands moves at constant angular velocities, the answer is ## \sqrt{7} .## But don't you think this assumption is somewhat doubtful and wrong?

Similar threads

  • · Replies 7 ·
Replies
7
Views
1K
  • · Replies 13 ·
Replies
13
Views
2K
Replies
17
Views
2K
  • · Replies 12 ·
Replies
12
Views
3K
  • · Replies 2 ·
Replies
2
Views
2K
  • · Replies 13 ·
Replies
13
Views
2K
  • · Replies 20 ·
Replies
20
Views
2K
Replies
3
Views
1K
  • · Replies 1 ·
Replies
1
Views
1K
  • · Replies 2 ·
Replies
2
Views
2K